NAVLE Question of the Day

¡Supera tus tareas y exámenes ahora con Quizwiz!

When a sow is giving birth, what is the shortest time interval between passage of piglets that indicates dystocia? A - 1 hour B - 1/2 hour C - 2 hours D - 4 hours E - 15 minutes

A - 1 hour Dystocia in the sow is present when one hour or more has passed between piglets. Intervention should be swift to prevent death of piglets from anoxia. Dystocia is not common in swine but when it occurs, it is often due to uterine inertia, similar to other animals that produce multiple offspring. Other causes include fetal malposition, fetopelvic disproportion, obstruction of the pelvic canal, deviation of the uterus, and excitement in the dam. Medical therapy can be given with oxytocin and/or calcium only when an obstructive dystocia is not present.

A client calls and says her cat was chewing on her lily plant two days ago and is now acting very sick. You tell her to bring the cat in immediately so that you can perform which of the following? A - BUN and creatinine levels B - Induce emesis and administer activated charcoal C - Abdominal radiographs to diagnose intestinal obstruction D - ALT, AST, GGT, and total bilirubin levels

A - BUN and creatinine levels Lily plant toxicosis is extremely serious and can cause rapid and fatal acute renal failure in cats. If ingestion is suspected, decontamination and aggressive fluid therapy, and monitoring of renal values are recommended immediately. In this case, since 2 days have passed, inducing emesis and administering activated charcoal would not be helpful. What you can do is check renal values (BUN and creatinine) and treat for acute renal failure. Ruling out an intestinal obstruction in a cat with this history is reasonable but not the best of the choices given.

A 4-year old male Manx cat presents to you because the owners found an empty, opened pill vial in the bathroom and the cat vomited. On physical exam, you note ptyalism and facial edema. The cat's mucous membranes are pale and slightly icteric. You perform a blood smear and detect Heinz bodies in erythrocytes. The cat's packed cell volume (PCV) is 26% (30-45%). The owners provide you a list of the medications in the medicine cabinet which are acetaminophen (Tylenol), finasteride (Propecia), enalapril (Vasotec), and omeprazole (Prilosec). What treatments should you institute for this cat? A - Acetylcysteine and S-adenosylmethionine B - Activated charcoal and whole blood transfusion C - Prednisone and amoxicillin D - Emesis and methylene blue

A - Acetylcysteine and S-adenosylmethionine Acetaminophen toxicity in cats usually occurs when owners administer the drug, unaware of its significant potential toxicity in cats. In this case, the cat's clinical signs are most consistent with acetaminophen toxicity based on the Heinz body anemia that is present. Cats can die from oxidative damage and methemoglobinemia within 1-2 days of ingestion. It may also be associated with hepatotoxicity in cats, although this is seen more frequently in dogs. Recall that cats are particularly sensitive to acetaminophen because they have decreased glucuronyl transferase activity which conjugates acetaminophen to glucuronic acid for excretion. As a result, 50-60 mg (a single tablet) may be fatal for a 4-5 kg cat. Treatment should consist of toxin removal if possible by inducing emesis in some cases. As the cat in this case is already vomiting, this may not be necessary. Activated charcoal is controversial and should only be given if ingestion occurred within hours and should be administered very carefully in cats due to the risk of aspiration. The specific antidote is acetylcysteine which binds to some of the reactive metabolites of acetaminophen and increases the availability and synthesis of glutathione. Other treatments may include S-Adenosylmethionine (SAMe) which has hepatoprotective and antioxidant properties. Cimetidine can be given to inhibit the p450 oxidase in the liver and limit formation of toxic metabolites. Ascorbic acid can also be used as an adjunct treatment to bind toxic metabolites. In cats with signs of hypoxemia from severe hemolytic anemia (PCV <20%), a transfusion and further supportive care may be warranted.

A 4 month old filly is presented with a 4 day history of lethargy, heavy breathing, cough and decreased appetite. On lung auscultation asymmetrically-distributed crackles and wheezes are audible. Some areas have no breath sounds and a dull resonance on thoracic percussion. A lateral chest radiograph shows consolidated nodular lung lesions and mediastinal lymphadenopathy. Pneumonia due to Rhodococcus equi infection is confirmed based on culture and polymerase chain reaction results on a transtracheal wash. Which one of following choices is the most appropriate treatment? Value Normal 104.9 F (40.5 C) 99.0-100.6, 37.2-38.1 C HR=72 bpm 28-40 bpm R=44 brpm 10-14 brpm A - Clarithromycin and Rifampin B - Chloramphenicol and Aminophylline C - Vancomycin and Atropine (if bronchospasm is observed) D - Procaine penicillin G and gentamicin E - Tilmicosin

A. Clarithromycin and Rifampin. In this case, a chest radiograph with consolidated nodular lung lesions and mediastinal lymphadenopathy is highly suggestive of Rhodococcus equi in a foal under 5 months of age. Combination treatment of rifampin combined with either erythromycin, clarithromycin, or azithromycin is appropriate to treat R. equi pneumonia. However, in one retrospective study, the combination of clarithromycin and rifampin was shown to be the most effective of the three. Tilmicosin, while a macrolide, is not very active against R. equi and swelling can occur at injection sites in foals; it is not recommended for treatment of R. equi pneumonia. If a foal under 5 months with signs of pneumonia had gram positive pleomorphic rods (like Chinese letters) in a transtracheal wash, these signs would also suggest R. equi. Polyarthritis, polysynovitis, and diarrhea can also be seen in affected foals. Click here to see an ultrasound video from Merck of a 3 month-old foal with rhodococcal pneumonia.

Which of these is the most common cause of seizures in the adult ferret? A - Hypoglycemia B - Idiopathic epilepsy C - Hypocalcemia D - Intracranial neoplasia E - Hepatic encephalopathy

A. Hypoglycemia Hypoglycemia secondary to insulinoma is the most common cause of seizures in the adult ferret. Keep in mind, a prolonged seizure can actually cause hypoglycemia, so the finding of low blood glucose in a seizuring ferret does not necessarily confirm a diagnosis of insulinoma. Idiopathic epilepsy has not been reported in ferrets. Hypocalcemia and hepatic encephalopathy can cause seizures, but are not as commonly reported as hypoglycemia.

Which of these are used in the minor cross match for blood products when looking for a compatible blood donor for a dog? A - Recipient red blood cells, donor plasma B - Donor red blood cells, recipient plasma C - Donor red blood cells, donor plasma D - Recipient red blood cells, recipient plasma

A. Recipient red blood cells, donor plasma In a minor crossmatch, you are looking to see if the factors in the plasma of the donor are going to react to the recipient's red blood cells.

A 4-year-old Standardbred mare is examined in the fall with a 5-day history of depression, left head tilt, facial paralysis and worsening gait. The horse is ataxic and knuckles on both forelegs, worse on the right. There is muscle atrophy of the left shoulder and right hindquarters and strips of localized spontaneous sweating over the left trunk. Which diagnostic test would best support the presumptive diagnosis? A - Serum:CSF antibody titer ratios for Sarcocystis neurona B - MRI to rule out nigropallidal encephalomalacia C - Serum IgM capture ELISA for West Nile Virus D - Serum Western blot analysis for eastern equine encephalomyelitis E - Plasma antibody test for St. Louis encephalitis

A. Serum:CSF Antibody titer ratios for Sarcocystis neurona Serum:CSF antibody titer ratios should be recommended to test for equine protozoal myeloencephalitis (EPM). Think of EPM due to to the Apicomplexan protozoa Sarcocystis neurona (or less commonly, Neospora hughesi ) when you see ataxia, asymmetrical muscle atrophy, and/or localized strips of sweating in horses, especially when neurologic signs are multifocal (i.e.:cranial (head tilt), spinal (foreleg ataxia)). Onset is typically insidious but can be acute and severe. Dx - Serologic tests including evaluation for surface antigens (snSAGs) and the indirect fluorescent antibody test (IFAT) show that a horse has been exposed to the organism. Positive serum results alone are difficult to interpret (can indicate exposure) but negative serum titers are often considered diagnostic except in very acute cases of disease. A ratio of serum to CSF antibody showing intrathecal antibody production against S. neurona combined with clinical suspicion is highly suggestive of a diagnosis of EPM. Definitive diagnosis requires post-mortem examination. If you see a febrile neurologic horse, think also of equine herpes myeloencephalopathy and the equine encephalidities-EEE, WEE and VEE, and West Nile virus encephalitis. Nigropallidal encephalomalacia is the result of poisoning with yellow star thistle.

A pregnant mare was brought out to your barn for observation in anticipation of parturition. After several hours of restless behavior, several gallons of allantoic fluid rush out from the vulva. Which of the following would you expect to happen next for a normal parturition? A - The thin, white, glistening amniotic membrane emerges from the vulva B - The placenta is expelled from the vulva C - The hind legs of the foal emerge from the vulva D - The red, velvety, chorioallantoic membrane emerges from the vulva

A. The thin, white, glistening amniotic membrane emerges from the vulva This case description is consistent with stage I of labor in the horse. The first stage of foaling typically lasts 30 minutes to 4 hours. During this stage, mares act restless and may exhibit signs similar to colic such as flank watching, pawing, and constantly getting up and down. When the placenta ruptures ("water breaks"), there may be several gallons of allantoic fluid that come out. Usually, within about 5 minutes, the second stage of labor begins and the foals feet and nose appear at the vulva, covered in the white, thin, glistening amnion. If a red, velvety, membrane is seen, this is the chorioallantois which indicates premature placental separation which can impair oxygen delivery to the fetus and can result in death of the foal. Usually, the muzzle will emerge from the amnion by the time the foal's hips pass through the pelvis but if not, the amnion can be gently broken and removed. Usually, the umbilical cord breaks naturally when the mare stands or foal begins to rise. Then, within 30 minutes to 3 hours after foaling, the placenta should be expelled.

What species is the principle carrier and vector of Malignant Catarrhal Fever (MCF) virus in North America? A - Donkey B - Sheep C - Cattle D - Bison E - Horse

B - Sheep MCF virus is ovine herpes virus-2 (OvHV-2) and is carried by 95-99% of sheep in North America which show no symptoms. It is also carried by 75% of domestic goats, 40% of muskox, 37% of bighorn sheep, 25% of pronghorn antelope, 62% of mouflon sheep, and by a small percentage of elk, mule deer, and white tailed deer. Susceptible hosts include cattle, water buffalo, deer, pigs, and bison; bison are the most susceptible.

A 4-year old male Manx cat presents to you because the owners found an empty, opened pill vial in the bathroom and the cat vomited. On physical exam, you note ptyalism and facial edema. The cat's mucous membranes are pale and slightly icteric. You perform a blood smear and detect Heinz bodies in erythrocytes. The cat's packed cell volume (PCV) is 26% (30-45%). The owners provide you a list of the medications in the medicine cabinet which are acetaminophen (Tylenol), finasteride (Propecia), enalapril (Vasotec), and omeprazole (Prilosec). What treatments should you institute for this cat? A - Emesis and methylene blue B - Acetylcysteine and S-adenosylmethionine C - Activated charcoal and whole blood transfusion D - Prednisone and amoxicillin

B - Acetylcysteine and S-adenosylmethionine Acetaminophen toxicity in cats usually occurs when owners administer the drug, unaware of its significant potential toxicity in cats. In this case, the cat's clinical signs are most consistent with acetaminophen toxicity based on the Heinz body anemia that is present. Cats can die from oxidative damage and methemoglobinemia within 1-2 days of ingestion. It may also be associated with hepatotoxicity in cats, although this is seen more frequently in dogs. Recall that cats are particularly sensitive to acetaminophen because they have decreased glucuronyl transferase activity which conjugates acetaminophen to glucuronic acid for excretion. As a result, 50-60 mg (a single tablet) may be fatal for a 4-5 kg cat. Treatment should consist of toxin removal if possible by inducing emesis in some cases. As the cat in this case is already vomiting, this may not be necessary. Activated charcoal is controversial and should only be given if ingestion occurred within hours and should be administered very carefully in cats due to the risk of aspiration. The specific antidote is acetylcysteine which binds to some of the reactive metabolites of acetaminophen and increases the availability and synthesis of glutathione. Other treatments may include S-Adenosylmethionine (SAMe) which has hepatoprotective and antioxidant properties. Cimetidine can be given to inhibit the p450 oxidase in the liver and limit formation of toxic metabolites. Ascorbic acid can also be used as an adjunct treatment to bind toxic metabolites. In cats with signs of hypoxemia from severe hemolytic anemia (PCV <20%), a transfusion and further supportive care may be warranted.

Which infectious agent causes the clinical presentation of pigs most similar to Haemophilus parasuis (Glasser's disease)? A - Mycoplasma hyosynoviae B - Streptococcus suis C - Fusobacterium necrophorum D - Erysipelothrix rhusiopathiae

B - Streptococcus suis Streptococcus suis and H. parasuis both cause polyarthritis, polyserositis, fever, and pneumonia in young piglets up to several weeks of age. Both can cause fibrinopurulent inflammation as well as meningitis and convulsions. Erysipelas and M. hyosynoviae typically occur in grower and finisher pigs and do not result in pneumonia. Diamond-shaped skin lesions (thus the name "diamond skin disease") are pathognomonic for Erysipelas. Fusobacterium necrophorum causes lameness via footrot or laminitis.

In September, 2 Quarter horse mares horses are presented that are pastured in a group of 5 in Oklahoma. The horses eat pasture grass supplemented by alfalfa hay and a small amount of grain. One mare was found peracutely dead this morning. The other is depressed, anorexic and colicy. Physical exam reveals dark, congested mucous membranes with small ulcer-like erosions. The mare makes frequent attempts to urinate, yielding red urine (hematuria) with a urine specific gravity (USG) of 1.006. She makes repeated attempts to drink small amounts of water and keeps her muzzle submerged in the water trough. Necropsy of the other mare shows that the stomach and bladder linings are irritated and hemorrhagic. What is the diagnosis? Value Normal 102.7 F (39.3 C) 99.0-100.6, 37.2-38.1 C HR=40 bpm 28-40 bpm BR=20 brpm 10-14 brpm A - Clostridium perfringens type A B - Cantharidin toxicity C - Sorghum cystitis D - Enzootic hematuria E - Arsenic toxicity

B. Cantharidin toxicity This is cantharidin toxicity caused by blister beetles (Epicauta spp.) which swarm in alfalfa hay during harvest. Cantharidin is a potent irritant: see colic, renal disease, hematuria, peracute death. Enzootic hematuria is a cow disease thought to be caused by bracken fern toxicity. In horses, bracken fern toxicity presents differently, causing neurologic disease. Expect thiaminase-related bracken STAGGERS in horses. When 20%-25% of the diet for 3 or more months is bracken fern, horses can exhibit anorexia, weight loss, incoordination, and a crouching stance with feet placed wide apart and an arched back and neck. Sorghum cystitis/ataxia is characterized by cystitis, urinary incontinence ("dribbling" ), posterior incoordination.

A one-year-old intact male domestic short hair cat is presented with dyspnea after vehicular trauma. On physical examination, the cat is open-mouth breathing with muffled heart sounds and borborygmi heard on pulmonary auscultation. Which one of the following choices is the most likely diagnosis? A - Cardiac tamponade B - Diaphragmatic hernia C - Posterior lung lobe torsion D - Flail chest E - Pneumothorax

B. Diaphragmatic hernia Diaphragmatic hernia is most likely to cause dyspnea with gastrointestinal sounds auscultable in the chest.

You see an 8-month old kitten with the effusive form of feline infectious peritonitis and perform euthanasia. The kitten was having severe diarrhea around the house when it became ill. The owner has a 2 year old cat at home and wants to know what this cat's prognosis is since it has been exposed to the sick kitten. Currently this cat is clinically healthy. What do you tell her? A - You recommend a coronavirus titer to determine it the cat is actively infected B - Feline infectious peritonitis is not contagious and because her other cat died of FIP does not mean this cat will succumb to the disease C- Perform a PCR on the cat's feces to see if the virus is being shed D - Her other cat may develop symptoms within the next two weeks because FIP is highly contagious E - Place the cat on L-lysine to prevent or suppress any infection with FIP

B. Feline infectious peritonitis is not contagious and because her other cat died of FIP does not mean this cat will succumb to the disease Feline infectious peritonitis is not a contagious disease. It is a disease that is caused by a mutation of feline enteric coronavirus. It is unknown why in some patients this virus mutates and causes the FIP syndrome. It is most likely to occur in young or immunocompromised cats. Her other cat is not necessarily going to get FIP just from exposure. In fact, the majority of the cat population has been exposed to the feline enteric coronavirus. Because most cats in the general population have been exposed, it makes interpretation of coronavirus titers difficult. The titers can be elevated due to prior exposure and not from FIP. The titers can only be interpreted in lieu of clinical signs, blood results, etc. L-lysine is an anti-viral medication that may have some benefit for suppression of herpes virus but would not be a prevention or treatment choice for coronavirus. The coronavirus is shed in the cat's feces during active infection with coronavirus. Some infected cats do not shed the virus. The virus attacks the intestinal tract and causes GI upset. PCR on the feces would detect coronavirus, but does not distinguish between the enteric coronavirus and the mutated FIP form of the virus.

Your client is pregnant and is worried about acquiring toxoplasmosis from her cat. What do you advise? A - Toxoplasmosis titer should be performed on the owner by a human physician. A positive titer indicates antibodies to the organism that will prevent infection in the first trimester. A negative titer indicates she should remove her cats from her environment. B - Submit toxoplasmosis titers from the cat. A positive cat infected with toxoplasmosis can shed multiple times in its lifetime and pose a zoonotic risk. C - Empty the litter box daily as a simple precaution to prevent infection as it takes 1-3 days for passed oocyts in the stool to sporulate into an infective form D - Submit toxoplasmosis titers from the cat. A IgG titer of 1:64 or greater suggests recent or active infection that could pose a danger.

C - Empty the litter box daily as a simple precaution to prevent infection as it takes 1-3 days for passes oocysts in the stool to sporulate into an infective form Toxoplasmosis gondii is a protozoal organism. The cat is the definitive host; the entire life cycle of the organism can be completed within this host. Most cats become infected when they consume an exposed rodent with bradyzoites encysted in their tissues. Only recently infected cats generally shed oocysts in their stool, and cats typically only shed these oocysts for 1-2 weeks. Most cats will only have one shedding episode in their lifetime. A IgM (not IgG) titer of 1:64 or greater suggests recent or active infection and that cat is at risk of shedding oocysts in their stools. Oocyts are not infective until they sporulate. This process takes > 24 hours, so emptying the litter box daily is advised, preferably by someone who is not pregnant. If an owner has owned cats for a long while, it is possible that they may have previously been exposed and therefore have mounted an immune response to the organism. If so, it may be advisable to test for Toxoplasma antibody titers in the owner. A sufficient antibody titer will mean the client is protected from infection during the first trimester.

A 2-year old male neutered cat presents to you depressed, hypersalivating, and ataxic with muscle tremors. The owner reports that a pyrethrin-based spot-on formulation for flea control belonging to their Golden Retriever was accidentally applied on the cat earlier today. Which of the following drugs will you use to treat the cat's clinical signs? A - Acepromazine B - Atropine C - Methocarbamol D - Amoxicillin E - 2-PAM

C - Methocarbamol Pyrethrins alter the activity of the sodium ion channels of nerves, which prolongs the period of sodium conductance. This increases the length of depolarization resulting in repetitive nerve firing. Cats are particularly sensitive to pyrethrin-containing products and can develop clinical signs within hours after administration. Affected animals should be bathed to remove remaining product. Minor clinical signs such as hypersalivation and ear twitching are usually self-limiting and do not require treatment. Control of marked tremors or seizures can be achieved with methocarbamol (Robaxin).

Which of these drugs is a neuromuscular blocker when given to a dog? A - Naloxone B - Edrophonium C - Atracurium D - Neostigmine E - Fentanyl

C. Atracurium The correct answer is atracurium. Atracurium is a non-depolarizing neuromuscular blocker. Other neuromuscular blockers include pancuronium, d-tubocurarine and succinylcholine. Succinylcholine is a depolarizing neuromuscular blocker. Fentanyl is an opioid. Naloxone is an opioid antagonist. Edrophonium and neostigmine are cholinesterase inhibitors used to reverse neuromuscular blockers.

What is the potential udder fate of goats infected with caprine arthritis encephalomyelitis virus? A - Gangrene B - The udder is not affected C - Hardbag D - Bluebag

C. Hardbag CAEV is a retrovirus and when it affects the udder it will cause fibrosis and result in a firm udder with agalactia. Treatment is ineffective and the goat should be culled. The disease is usually subclinical but can cause arthritis in adults and encephalitis in kids.

Which of the following pairs of tissues can both cause increased serum alanine aminotransferase (ALT) levels in dogs and cats? A - Pancreas and intestine B - Kidneys and pancreas C - Liver and muscle D - Spleen and liver E - Heart and kidneys

C. Liver and muscle Damage to liver and muscle cells of dogs and cats causes increased serum levels of alanine aminotransferase (ALT). ALT is a considered a "leakage" enzyme. High levels are normally found in the cytoplasm of healthy cells. When hepatic or muscle cells are damaged, ALT leaks into adjacent tissue where it is picked up by the venous circulation. Other animals such as horses, ruminants, pigs, and birds do not have high levels of ALT inside cells. Aspartate aminotransferase (AST) is the leakage enzyme predominant in these animals. The magnitude of the elevation of leakage enzymes can be deceiving - are a few cells leaking a lot or many cells are leaking a small amount? Severe damage to a healthy liver may result in very high levels, while low levels may be seen when significant atrophy or fibrosis of the liver is present and few cells are left. Prognosis of the former may be good, for the latter it is certainly grim. Differentiation between muscle and liver as the source of increases in ALT/AST is determined by evaluation of other muscle (creatine phosphokinase (CK) and AST) and liver enzymes (AST, sorbitol dehydrogenase (SDH), gamma glutamyl transferase (GGT), alkaline phosphatase (ALP)), the animal, and the clinical evaluation. Leakage enzymes do NOT provide an estimate of actual liver function. This is evaluated by measurement of substances dependent on the efficacy of liver function such as bile acids, blood urea nitrogen, albumin, and glucose.

An adult horse with clinical signs of voluminous gastric reflux, depression, colic, and fever is most likely suffering from which condition? A - Ulcerative duodenitis B - Proliferative enteropathy C - Proximal enteritis D - Right dorsal colitis E - Cantharidin toxicity

C. Proximal enteritis Proximal enteritis, or duodenitis-proximal jejunitis, is a clinical syndrome characterized by large volumes of gastric reflux resulting from excessive fluid and electrolyte secretion into the small intestine and small intestinal inflammation and edema. Laminitis is an important potential sequela. The cause is unknown but several bacteria and toxins including Clostridium difficile, Clostridium perfringens, Salmonella, and fumonosin B1 mycotoxins have been implicated. Ulcerative duodenitis is a disorder of foals resulting in fever, colic, diarrhea, and delayed gastric emptying. Lawsonia intracellularis causes proliferative enteropathy in foals and weanlings, a disease characterized by hypoproteinemia, diarrhea, chronic ill thrift, and ventral edema. Right dorsal colitis is typically a result of non-steroidal anti-inflammatory drug toxicity and results in hypoproteinemia and colic. Cantharidin toxicity (blister beetles, Epicauta spp.) causes a wide range of clinical signs predominated by profuse diarrhea, stranguria and pollakiuria, and colic.

What is the most common neoplasia seen in the equine stomach? A - Lymphosarcoma B - Mesothelioma C - Squamous cell carcinoma D - Gastric adenocarcinoma

C. Squamous cell carcinoma

Severe combined immunodeficiency is a lethal autosomal recessive trait in Arabian foals. Heterozygotes are clinically normal. If the heterozygote carrier rate for the genetic mutation is 8%, what is the expected frequency of Arabian foals that are homozygous for the mutated allele? A - 25% B - 4% C - 0.064% D - 0.16% E - 2% F - 0.64%

D - 0.16% You do not need to know anything about the disease in this question in order to get the correct answer. You are told that the disease is recessive and has a carrier rate of 8% Because the trait is recessive, homozygotes will be the only individuals affected. In order for a foal to be born homozygous for the trait, BOTH parents MUST be carriers. The chances of both parents being carriers is 8% x 8% (or 0.08 x 0.08) = 0.0064 or 0.64%. If both parents are carriers, the offspring has a 1 in 4 chance of inheriting two mutant alleles (50% chance for each allele from each parent). Since the chances of both parents being carriers is 0.64% and the chance of having a homozygous offspring in that case is 1 in 4, the overall expected frequency of diseased foals is 0.0064 x 0.25= 0.0016 or 0.16%.

A 2 year old Angus cow is presented with weakness of the hind limbs and a staggering gait. On physical exam, the heart rate and respiratory rate are slow and irregular, and the pupils are dilated. The cow's breath and urine smell like the odor of mouse urine. Suddenly the cow's pulse becomes rapid and thready, she collapses, and dies of apparent respiratory failure. Which one of the following plants is most likely to have caused this spectrum of clinical signs? A - Pinus ponderosa (Western yellow pine) B - Veratrum spp (False hellebore, Skunk cabbage) C - Centaurea spp (Knapweed, Yellow star thistle) D - Conium maculatum (Poison hemlock) E - Tetradymia spp (Horsebrush)

D. Conium maculatum (poison hemlock) Poisoning with Conium maculatum (Poison hemlock) is responsible for the death of this cow. At least 8 toxic piperidine alkaloids have been isolated from poison hemlock. Coniine is found in seeds and the mature plants; g-coniceine is found in young growing plants. Poison hemlock is toxic to all livestock and humans. Signs of toxicity develop 1-2 hours after ingestion and are usually fatal. Signs include nervousness, trembling, weakness especially of the hind limbs, weak pulse, irregular heart rate, recumbency, coma and death. A mousy odor exuding from the urine and breath is pathognomonic. Ingestion of poison hemlock during gestation causes arthrogryposis and other congenital defects in cattle, goats, and pigs.

When performing a subtotal colectomy on a feline patient, what blood vessel limits the amount of colon that you are able to remove? A - Caudal mesenteric artery B - Pudendal artery C - Left colic artery D - Ileocolic artery

D. Ileocolic artery The ileocolic artery provides blood supply to the ascending and transverse colon. The site for colonic resection is limited by tension on the ileocolic artery when trying to suture your new end of colon to the rectum. Sometimes the tension is too great and instead of a colocolic anastomosis, an ileocolic anastomosis must be performed. Essentially you are trying to connect a section of ascending colon to the rectum. Performing an ileocolic anastomosis is not ideal because you eliminate the ileocecal valve, and that may predispose the animal to bacterial overgrowth. The caudal mesenteric artery gives branches to the rectum and descending colon. The left colic artery also feeds the descending colon. The pudendal artery supplies the external genitalia.

A one-month old foal develops fever, icterus, and diarrhea acutely. Bloodwork shows hyperfibrinogenemia, hypoglycemia, and elevated liver enzymes. Which of these conditions is most likely? A - Theiler's disease B - Clostridium novyi type B C - Herpesviral hepatitis D - Tyzzer's disease

D. Tyzzer's disease This is the most likely cause because of the age of the foal and the acute nature of the disease. Tyzzer's disease is caused by Clostridium piliformis, which causes an acute necrotizing hepatitis. It affects only foals from about 1-6 weeks of age. Theiler's disease is a condition of adult horses. Clostridium novyi is rare in horses and seen much more in sheep and cattle. Herpesvirus can cause hepatitis but is usually seen at or very soon after birth.

A horse is suspected of having the muscle disease rhabdomyolysis, and you are seeking a laboratory test to help confirm the diagnosis. Of the following, which test would likely be the most helpful? A - Gamma glutamyl transferase (GGT) B - Ionized phosphate C - Anion gap D - Sorbitol dehydrogenase (SDH) E - Creatine kinase (CK)

E - Creatine kinase (CK) CK, also called, CPK, is found in muscle cells and is elevated in the serum when muscle damage occurs.

Which of these drugs has the greatest potential for causing acute renal failure in the horse? A - Oxytetracycline B - Dexamethasone C - Xylazine D - Diphenhydramine E - Neomycin

E - Neomycin The correct answer is neomycin. Aminoglycosides are one of the most common causes of renal tubular nephrosis and acute renal failure. Of the aminoglycosides, neomycin is probably the most nephrotoxic, followed by gentamicin, amikacin, and streptomycin. The other big class of nephrotoxic drugs is non-steroidal anti-inflammatory drugs.

Which one of the following is the most common cause of dystocia in cattle? A - Uterine inertia B - Fetal monsters C - Maternal birth canal abnormalities D - Abnormal fetal presentation, position, and/or posture E - Fetopelvic disproportion

E. Fetopelvic disproportion Fetopelvic disproportion, where the fetus is too large to pass through the maternal pelvic canal, is the most common cause of dystocia in cattle. It is a significant cause of calf losses, especially in first-calf heifers. Fetopelvic disproportion may occur in small heifers with normal sized calves or in cows with large fetuses. The incidence of dystocia in heifers increases as the age and size of replacement heifers decreases. The size of the sire and other factors also contribute to the incidence of dystocia. Close monitoring of heifers and early intervention are necessary to prevent calf loss. Although Stage II labor in cows can be longer than in horses, when heifers have difficulty moving a calf for more than 30 minutes, it is time to call for assistance.

Which choice is a competitive antagonist for aldosterone that is sometimes used in treatment of congestive heart failure? A - Thiazides (chlorothiazide, hydrochlorothiazide) B - Furosemide C - Osmotic diuretics (Mannitol, DMSO) D - Carbonic anhydrase inhibitors (Acetazolamide) E - Potassium-sparing diuretics (Spironolactone, Amiloride)

E. Potassium-sparing diuretics (Spironolactone, Amiloride) Potassium-sparing diuretics (spironolactone, amiloride) are aldosterone antagonists sometimes used with furosemide as part of the treatment plan for congestive heart failure (CHF). Aldosterone, the mineralocorticoid produced in the adrenal cortex, acts on the distal convoluted tubules (and collecting ducts) of the nephron to retain Na+ and water, secrete K+ and increase blood pressure. Clinically, look for increased aldosterone secretion (increased blood pressure) with CHF. In contrast, see decreased aldosterone secretion (and decreased blood pressure) with hypoadrenocorticism.

A herd of ewes has had an abortion storm in which 80% of animals are affected. All stages of gestation are affected, and lambs that are born are very weak. On the placentas, cotyledons have white foci of necrosis. Which one of the following choices is the primary differential diagnosis? A - Listeria monocytogenes B - Brucella C - Chlamydiosis D - Histophilus ovis E - Toxoplasma gondii

E. Toxoplasma gondii Toxoplasma gondii infection can cause severe abortion storms in sheep. Clinical signs pathognomonic of T. gondii are bright to dark red cotyledons with white foci of necrosis. The intercotyledonary areas of placenta are normal. With Brucella and Histophilus ovis the abortion rate is much lower. Abortion caused by Listeria spp. is associated with the feeding of silage.

What is a common side effect of xylazine administration in cats? A - Vomiting B - Seizures C - Anuria D - Polycythemia E - Miosis

A - Vomiting The correct answer is vomiting. Xylazine frequently causes vomiting in cats. In fact, veterinarians use Xylazine when they wish to induce emesis in cats. It can also cause decreased PCV, mydriasis, and diuresis.

A horse presents to you for chronic, recurrent laminitis and skin disease. You notice on your exam that the horse has a particularly thick, long, wavy, and matted coat. The owner mentioned that this developed many months ago. What is a likely diagnosis? A - Cushing's disease (Pituitary Pars Intermedia Dysfunction) B - Hypothyroidism C - Pheochromocytoma D - Diabetes insipidus

A - Cushing's disease (Pituitary Pars Intermedia Dysfunction) The correct answer is Cushing's disease. The coat condition described is what horses with glucocorticoid excess develop; it is referred to as hirsutism. They will also be predisposed to infections including laminitis and skin diseases such as Dermatophilus. They are also frequently polyuric, polydipsic, and polyphagic.

A 1-year old female spayed Doberman Pinscher has presented after being hit by a car. Initial chest radiographs show mild contusions, and the patient appears to be otherwise stable. A right mid-shaft long oblique femoral fracture has been identified. Routine pre-operative blood work is unremarkable. A buccal mucosal bleeding test (BMBT) is elevated at 6 minutes. What will you administer prior to surgery? A - Desmopressin acetate B - 1,25 dihydrocholecalciferol C - Vitamin K D - Whole blood transfusion

A - Desmopressin acetate The correct answer is desmopressin acetate (DDAVP). Administration of desmopressin results in release of von Willebrand factor, which will help this patient with clotting. Given this dog's breed and elevated BMBT there is a very strong likelihood she is afflicted with von Willebrand's disease. In Dobermans this results in an inability to form a clot. This can be life threatening if the dog is taken to surgery. A whole blood transfusion does not provide an adequate source of von Willebrand factor but may be necessary if the patient's bleeding cannot be controlled despite appropriate pre-operative measures. 1,25 dihydrocholecalciferol is the active form of vitamin D which aids intestinal resorption of calcium. The BMBT does not assess factors 2, 7, 9, or 10 and therefore vitamin K is not indicated.

A 1.5-year old Quarter Horse gelding is presented to you for symmetric ataxia, weakness, and spasticity of all limbs, but worse in the hind limbs. When walking, the horse frequently drags his toes and the hind limbs frequently interfere with one another. Based on the signalment, history and physical examination findings, which of the following is the most likely cause of these clinical signs? A - Equine Degenerative Myeloencephalopathy (EDM) B - Botulism C - Equine Protozoal Myeloencephalitis (EPM) D - Equine Motor Neuron Disease (EMND) E - Cauda Equina Syndrome

A - Equine Degenerative Myeloencephalopathy (EDM) The horse in this question has clinical signs most consistent with EDM; cervical vertebral malformation (wobblers) is also a possibility, but was not provided as an answer. The cause of EDM is unknown, but this disease typically affects young horses (< 2-3 years of age; but older horses can develop disease). Clinical signs are a result of diffuse neuronal fiber degeneration of various portions of the central nervous system. This disease has been associated with low serum vitamin E concentrations, suggesting that oxidative damage may play a role in the development of disease. EMND is typically associated with muscle tremors, shifting of weight while standing, muscle atrophy and recumbency. Botulism is associated with generalized muscle weakness. Cauda equina syndrome causes analgesia of the perineum. EPM can cause a range of clinical signs, but is typically with asymmetric neurologic deficits.

A 4 -year old domestic short haired cat presents for anorexia and weight loss of 1 week. Physical exam reveals a body condition score of 7/9, jaundice of the skin and sclera, and dehydration of 4%. Temperature is normal. Bloodwork shows: ALT=303 (25-97 U/L) GGT=1.8 (0-6 U/L) ALP=1170 (0-45 U/L) bilirubin=3.0 (0-0.1 mg/dl) Radiographs show an enlarged liver. What is the most important treatment for the cat's likely diagnosis? A - Esophagostomy tube feeding B - Oral S-adenosylmethionine for at least 1 month C - Oral clavulanic acid and amoxicillin for 4 weeks D - Parenteral vitamin K injections

A - Esophagostomy tube feeding The cat described likely has hepatic lipidosis. Cats that are greater than 2 years of age and obese are at the greatest risk for hepatic lipidosis. Often these cats are indoor-only and have had a recent stress in their life. An obese cat that is not eating with the above symptoms is most likely to have hepatic lipidosis. An ALP elevation that greater in magnitude than GGT is also suggestive of hepatic lipidosis. A bile duct obstruction, cholangiohepatitis, lymphoma, and FIP are other differentials but are less likely with the given information.

What is the potential udder fate of goats infected with caprine arthritis encephalomyelitis virus? A - Hardbag B - Gangrene C - The udder is not affected D - Bluebag

A - Hardbag CAEV is a retrovirus and when it affects the udder it will cause fibrosis and result in a firm udder with agalactia. Treatment is ineffective and the goat should be culled. The disease is usually subclinical but can cause arthritis in adults and encephalitis in kids. For clinical cases, the most common presentation is chronic arthritis (*REPORTABLE*). It can also cause progressive paresis (more common in kids), pneumonia, mastitis, and weight loss. It is transmitted mainly in colostrum and milk, but horizontal transmission can occur.

What is the primary mode of transmission of West Nile Virus (WNV) between birds? A - Via mosquitoes B - Via saliva C - Via Cnemidocoptes mites D - Via placenta E - Via feces

A - Via mosquitoes Mosquito vectors are the primary mode of transmission for WNV between birds and other hosts (horses, humans). There are infrequent documented cases of the disease being spread by feces or saliva. No mites have been documented to transmit the disease. Birds do not have a placenta

A chicken operation has recently been ravaged by a respiratory disease affecting almost all of the chickens in the flock. The chickens are coughing and sneezing and many have facial swelling. You necropsied many of the chickens and found mucoid exudate in the bronchi, thickened air sacs, and in a few of the chickens, interstitial nephritis was present. Which of these diseases is likely? A - Infectious bronchitis B - Infectious bursal disease C - Aspergillus D - Fowl cholera

A - Infectious bronchitis This is caused by a coronavirus. It is spread by aerosol and ingestion and usually affects all exposed birds. The clinical signs and necropsy findings are as described in the question. The disease can be clinically indistinguishable from mild forms of Newcastle disease, laryngotracheitis, and infectious coryza. Virus isolation is needed to obtain a definitive diagnosis. Aspergillus will cause fibrinonecrocaseous granulomatous air sacculitis, pneumonia, and tracheitis. Fowl cholera is caused by Pasteurella multocida. May see septicemia and death in large numbers or more chronic presentations (fibrinoheterophilic exudate in wattles, joints, hocks, and egg yolk peritonitis).

Two bucket-fed veal calves are depressed and stunted and have sticky feces coating their hindquarters. Fluid-splashing sounds are audible during auscultation over the left flank when a calf is drinking. Passage of a stomach tube obtains a rancid-smelling liquid with a pH of 5.2 [N=5.9-6.2] Which one of the following choices is the most appropriate treatment? A - Remove fermented rumen contents, flush with saline B - Wean affected calves; Shift to bottle-feeding for well calves C - Vitamin E/Selenium injections, plus dietary supplements D - Cull affected calves E - Inoculate rumen fluid from a healthy cow into calf by stomach tube

A - Remove fermented rumen contents, flush with saline. This is the clinical picture of a ruminal drinker calf with chronic indigestion from milk deposited directly into the rumen. Treat by removing the fermented material and flushing the rumen with saline. Attempt to induce gastric/reticular groove closure during feeding by inducing vigorous sucking activity with a finger before feeding milk. If calves relapse, consider weaning. The problem occurs due to failure of the gastric groove reflex, which normally shunts milk directly into the abomasum, (bypassing the rumen). Most often seen in bucket fed calves who gulp their milk.

A person calls to to say she adopted a friend's dog and requests a copy of the dog's medical record and radiographs. The former owner is a client at your clinic. Which one of the following choices is the most appropriate action to take? A - Require original owner consent or court order to release information B - Give caller a copy of medical record only C - Give caller a copy of medical record and radiographs D - After physical exam, start a new medical record documenting previous issues E - Give a summarized record with personal identifiers (vet, owner names) blocked out

A - Require original owner consent or court order to release information According to the AVMA's Principles of Veterinary Medical Ethics (2008 revision) "Ethically, the information within veterinary medical records is considered privileged and confidential. It must not be released except by court order or consent of the owner of the patient. Veterinarians should secure a written release to document that request."

You are presented a healthy 10 yr old maiden mare for breeding evaluation in January. On rectal palpation, both ovaries have multiple, small (less than 15 mm) follicles and no corpus lutea. The uterus is flaccid with no edema. Which one of the following choices is the most likely explanation of the findings in this mare? A - Seasonal anestrus B - The mare is in diestrus C - Estrus has just passed D - Early pregnancy E - The mare is too old and now is infertile

A - Seasonal anestrus Mares are seasonal breeders and enter anestrus in the short day months, late fall and winter. In anestrus the ovaries are inactive with small follicles and no corpus lutea. During estrus, the uterus softens and multiple follicles form on the ovaries. During pregnancy the uterus has good tone.

Fractures of the proximal sesamoid bones in horses are often associated with damage to which structure? A - Suspensory ligament B - Superior check ligament C - Deep digital flexor tendon D - Superficial digital flexor tendon E - Impar ligament

A - Suspensory ligament The suspensory ligament is most likely to be damaged with fracture of the proximal sesamoids due to its insertion onto these bones. Proximal sesamoid fractures are relatively common, caused by overextension. The prognosis for return to soundness is often predicated on the extent of the damage to the suspensory apparatus. Proximal sesamoid fractures and their attendant suspensory apparatus problems are the number one cause of racetrack deaths. If you have trouble visualizing the anatomy involved in proximal sesamoid fractures, there are excellent pictures and a clinical summary in Guide to Equine Clinics: LAMENESS vol. II, pp. 116-117, Pasquinis, Jahn & Bahr.

A 3-year old, indoor-only, male castrated Maine Coon cat is presented to you for acute-onset of hindlimb pain and paresis. Physical examination reveals a grade III left parasternal heart murmur and minimal movement in the hind legs. Femoral pulses are bilaterally absent, and the hindlimb toes are cold to the touch. The cat is tachypenic, but lung sounds are normal. What is the most likely diagnosis? A - Thrombus at the aortic bifurcation (saddle thrombus) B - T3-L3 intervertebral disc disease C - Lymphoma of the spinal cord D - Chlorpyrifos toxicity E - Tetanus

A - Thrombus at the aortic bifurcation (saddle thrombus) Maine Coon cats are predisposed to development of hypertrophic cardiomyopathy at a young age. Consequently, left atrial enlargement predisposes to atrial thrombus formation, and these clots frequently lodge in the arterial supply to the hindlimbs. The trifurcation is where the aorta divides into the two external iliac arteries and the common origin of the internal iliac arteries. Classic findings due to a clot at the aortic trifurcation include posterior paresis/paralysis, hindlimb pain, cyanotic nailbeds, absent femoral pulses, and a firm leg musculature. Other signs of cardiac disease/failure (murmur or pulmonary edema) are often but not always evident at presentation. Neither cord lesions, tetanus, nor toxicities should cause the vascular compromise evident on this cat's physical exam.

An aged intact female potbellied pig presents for evaluation of abdominal distension. A large mass contiguous with the uterine wall is discovered with ultrasound exam of the abdomen. Which of the following choices is the most likely cause of this finding? A - Uterine leiomyoma B - Endometritis C - Adenocarcinoma of the cervix D - Lymphosarcoma E - Endometrial hyperplasia

A - Uterine leiomyoma The most likely cause of a large uterine mass in an intact female potbellied pig is a uterine leiomyoma. These tumors do not typically metastasize so a cure may be achieved by surgical removal of the utuers.

You serologically test 100 Siberian box turtles for galloping halitosis. 27 turtles test seropositive and 73 test seronegative. However, molecular testing reveals 3/27 of the seropositive turtles are disease free and 10/73 of the seronegative turtles are diseased. The entire fur-bearing turtle industry depends on your answer: What is the Predictive Value Positive (PVP) of your serologic test? A - 95% B - 89% C - 86% D - 73% E - 70%

B - 89% Remember - you are comparing *TWO TESTS* here. PVP means "of the turtles *my test* says are positive (27), how many are truly positive?" (27-3=*24*, this goes in the "a" box). Here is how you do it: first draw a 2x2 table, and label the boxes a, b, c, and d. PVP = a/(a+b). Now, add in the TOTAL number of animals (100), the total positive by YOUR test (27) and the total negative by YOUR test (73). Now the (slightly) tricky part. Add in the numbers that YOUR test got WRONG according to the gold standard test. (3 false positives in box b, 10 false negatives in box c). Last, subtract to fill in your "d" box (73-10=63) and do the math to calculate PVP = a/(a+b) = 24/27 = 0.89 = 89%. FYI: You can calculate sensitivity a/(a+c), specificity b/(b+d). PVP a/(a+b) and PVN d/(c+d).

Which one of the following conditions occurs in the Fall in temperate climates, among cattle moved from poor pasture to lush pasture? A - Rumenal hyperkeratosis B - Acute bovine pulmonary emphysema and edema C - Vena caval thrombosis and metastatic pneumonia D - Verminous bronchitis (Husk) E - Vagal indigestion

B - Acute bovine pulmonary emphysema and edema Acute bovine pulmonary emphysema and edema (ABPEE) is associated with Fall onset, occurring 5-10 days after changing cattle from poorer to better, often lush, pasture. Toxicity occurs because rumen micro-organisms convert L-tryptophan in lush pasture grasses to pneumotoxic 3-methylindole, which damages respiratory epithelial cells. Moldy sweet potato and Purple mint can cause a clinicopathologic syndrome that is indistinguishable from ABPEE. The toxic compounds are respectively, 4-Ipomeanol (Moldy Sweet Potato) and Perilla ketones (Purple Mint Toxicity).

A 4-year old female ferret presents with a history of an enlarged vulva and truncal alopecia. What is the most likely diagnosis? A - Insulinoma B - Adrenal tumor C - Sertoli cell tumor D - Hypothyroidism

B - Adrenal tumor The correct answer is adrenal tumor. These tumors are very common in middle-aged ferrets and usually secrete estradiol. Common clinical signs are hair loss, enlarged vulva, pruritus, and behavior changes. Diagnosis is made by clinical signs, ultrasound, and measurement of serum estradiol levels. Treatment is adrenalectomy. Insulinomas occur in midd-aged to old ferrets. There is typically a solitary raised pink nodule. Hyperestrogenism is a differential for adrenal gland tumors. This is typically seen in young jills (1-2yrs). They present with a swollen, fleshy vulva, bilateral symmetric flank/tail alopecia, lethargy, inappetence, and weakness.

A three year old male neutered Weimaraner dog is presented with a two day history of worsening anorexia, lethargy, cough and exercise intolerance. On physical exam, the mucous membranes are pale with a few petechiae. T=99.8 F (37.7 C)..[N=99.5-102.5 F, N=37.2-39.2 C] RR=24 brpm.....[N= 15-34] HR=144 bpm....[N= 60-120] A coagulation profile shows the following: Thrombocytes= 343,500 per microliter..[N=200,000-900,000] Buccal mucosal bleeding time (BMBT), normal Activated partial thromboplastin time (aPTT), increased Prothrombin time (PT), increased Thrombin time (TT), normal Fibrin degradation products (FDPs), normal Which one of the following choices is the most appropriate diagnosis? A - Disseminated intravascular coagulation (DIC) B - Anticoagulant rodenticide toxicity C - Von Willebrand's disease D - Idiopathic thrombocytopenia E - Neonatal isoerythrolysis

B - Anticoagulant rodenticide toxicity Anticoagulant rodenticide toxicity (or liver disease) can demonstrate increased activated partial thromboplastin time (aPTT) and prothrombin time (PT). Sometimes a mild to moderate thrombocytopenia may occur (50,000-150,000/?l). Proteins induced by vitamin K1 absence or antagonism (PIVKA) will be prolonged. The PIVKA test is more sensitive than PT/aPTT but is not as widely available. A threefold increase in PT or PIVKA is supportive of anticoagulant rodenticide toxicosis. Often there is no history of exposure to a rodenticide. Clinically, vague initial clinical signs like anorexia, lethargy and exercise intolerance may be the first manifestation of illness. With progression, you may see an acute hemorrhagic presentation. BMBT is an in vivo test to screen for platelet function. This test should not be performed on dots with abnormal platelets counts and clotting times. Prolonged BMBTs indicate von Willebrand's disease, thrombocyopathia, severe azotemia, and aspirin/NSAIDs. FDPs indicate plasmin activity by looking at fibrinogen and soluble and cross-linked fibrin. Increases are often associated with DIC.

What is a dysmature foal? A - Born post-term, normal axial skeletal size but emaciated B - Born post-term but small C - Born pre-term, but with paradoxically normal respiratory function D - Born to mare that ate endophyte-infested fescue E - Born pre-term with ciliary dyskinesia

B - Born post-term but small Also called "small for gestational age", these foals are thought to have suffered placental insufficiency in-utero. A postmature foal is a post-term foal that with normal axial skeletal size but is thin to emaciated. Most commonly born to mares ingesting endophyte-infested fescue. Gestation length in mares ranges from 310 days to more than 370 days. Typical length is 340 days.

A 1.5-year old Quarter Horse gelding is presented to you for symmetric ataxia, weakness, and spasticity of all limbs, but worse in the hind limbs. When walking, the horse frequently drags his toes and the hind limbs frequently interfere with one another. Based on the signalment, history and physical examination findings, which of the following is the most likely cause of these clinical signs? A - Equine Protozoal Myeloencephalitis (EPM) B - Equine Degenerative Myeloencephalopathy (EDM) C - Equine Motor Neuron Disease (EMND) D - Cauda Equina Syndrome E - Botulism

B - Equine Degenerative Myeloencephalopathy (EDM) The horse in this question has clinical signs most consistent with EDM; cervical vertebral malformation (wobblers) is also a possibility, but was not provided as an answer. The cause of EDM is unknown, but this disease typically affects young horses (< 2-3 years of age; but older horses can develop disease). Clinical signs are a result of diffuse neuronal fiber degeneration of various portions of the central nervous system. This disease has been associated with low serum vitamin E concentrations, suggesting that oxidative damage may play a role in the development of disease. EMND is typically associated with muscle tremors, shifting of weight while standing, muscle atrophy and recumbency. Botulism is associated with generalized muscle weakness. Cauda equina syndrome causes analgesia of the perineum. EPM can cause a range of clinical signs, but is typically with asymmetric neurologic deficits.

What is a gilt? A - Castrated male sheep B - Female pig that has not had a litter yet C - A male pig, castrated before puberty D - Ewe that has lambed at least once E - Female pig that has had more than one litter

B - Female pig that has not had a litter yet A young female pig that has not had a litter yet is called a gilt. A male pig, castrated before puberty is called a barrow. A wether is a castrated male sheep. FYI-A female ferret is called a Jill and a male is called a Hob.

What tumor type is a cat predisposed to developing at vaccination sites? A - Squamous cell carcinoma B - Fibrosarcoma C - Lymphoma D - Melanoma

B - Fibrosarcoma The correct answer is fibrosarcoma. Vaccine-associated fibrosarcomas are very well documented in cats.

What profile would you expect in a dog with hypervitaminosis D? A - High Ca, Low P B - High Ca, High P C - Low Ca, High P D - Low Ca, Low P

B - High Ca, High P Excessive intake of vitamin D is associated with an increase in 25-hydroxyvitamin D3 levels. At high levels, 25-hydroxyvitamin D3 competes with 1,25-dihydroxyvitamin D3 for its receptors on the intestines and bone causing increased absorption of Ca and P from the intestinal tract and resorption of bone causing increased levels of circulating Ca and P. A common source of confusion is that this is in contrast to PTH which causes high Ca but generally causes unchanged or normal phosphorus because it also enhances renal phosphorus excretion.

What profile would you expect in a dog with hypervitaminosis D? A - Low Ca, Low P B - High Ca, High P C - Low Ca, High P D - High Ca, Low P

B - High Ca, High P Excessive intake of vitamin D is associated with an increase in 25-hydroxyvitamin D3 levels. At high levels, 25-hydroxyvitamin D3 competes with 1,25-dihydroxyvitamin D3 for its receptors on the intestines and bone causing increased absorption of Ca and P from the intestinal tract and resorption of bone causing increased levels of circulating Ca and P. A common source of confusion is that this is in contrast to PTH which causes high Ca but generally causes unchanged or normal phosphorus because it also enhances renal phosphorus excretion.

You are asked to examine some feeder pigs that have stopped eating yesterday. The group is lying down and seems lethargic. They have fevers of 105-106F (40.6 -41.1 C), firm dry feces, and the skin has rhomboid-shaped red blotches scattered on it. What treatment should be recommended? A - Streptomycin B - Penicillin C - Metronidazole D - Gentamicin E - Chloramphenicol

B - Penicillin Erysipelas is susceptible to penicillins, as well as tetracyclines (usually), lincomycin and tylosin. Chloramphenicol and nitroimidazoles (including metronidazole) are not approved for food animal use. Gentamicin and Streptomycin are aminoglycosides which is only effective against gram-negative bacteria.

A 2-year old female DSH cat presents for weight loss, anorexia, dyspnea, and lethargy. She was previously treated with antibiotics but is still febrile on physical exam. You detect pleural effusion and notice that the abdomen is distended. On CBC there is a non-regenerative anemia, neutrophilia, and lymphopenia. On chemistry there is hyperproteinemia and a slight elevation in liver enzymes. What is your primary differential? A - Feline immunodeficiency virus B - Feline calicivirus C - Feline infectious peritonitis D - Feline leukemia virus

C - Feline infectious peritonitis The correct answer is FIP. This poor cat is infected with the dreaded wet form of feline infectious peritonitis. Good job if you nailed it! If you think about the pathogenesis of the virus, then these clinical signs make a lot of sense. With FIP, the classical lesion is pyogranulomatous vasculitis due to antigen-antibody complexes depositing in the venular endothelium, which results in pleural and peritoneal effusion. The CBC findings are common for FIP but not too specific. In greater than 50% of cats with the wet form of FIP, there will be hyperproteinemia. In greater than 70% of cats with the dry form of FIP there is hyperproteinemia.

A 2-year old nonlactating dairy cow in California has a sudden onset of head tilt and drooling. The owner also reports that she appears less active and less interested in feed than normal, and today is circling in one direction in the pen. You examine her and find T=105F (40.6 C), HR=96, and RR=32. There is ptosis, drooped ear, and weakness of the lips on the affected side. You take a lumbosacral spinal tap (see photo), and submit the CSF to your lab. The results show elevated protein and WBCs, with the cell type being mainly monocytes. The lab reports seeing some gram positive bacteria in the monocytes of the CSF. Based on these findings what is the best treatment for this condition? A - Chloramphenicol B - Metronidazole C - Penicillin D - Enrofloxacin E - Metoclopramide

B - Penicillin The diagnosis is Listeriosis. Listeria monocytogenes can effectively be treated in the early stages of the disease with penicillin, ampicillin, or tetracycline. Intramuscular procaine penicillin for example has a withdrawal of 10 days for slaughter (meat) and 48 hours for milk if the animal is lactating. Other choices of approved antimicrobials would also likely be effective since L. monocytogenes is susceptible to most antimicrobials. For a list of approved animal drugs see www.farad.org/vetgram. FARAD is the United States Food Animal Residue Avoidance Databank which gives withdrawal information as well as a list of prohibited drugs. The other drugs listed as choices here are prohibited and cannot be legally used in food animals in the USA.

A 2-month old calf is presented that was found dead. The night before this calf was weak, feverish and had a mucoid brown diarrhea with a small amount of blood and a nasty odor. Necropsy reveals a pseudo-diptheritic membrane lining parts of the distal small bowel and large bowel. What is the diagnosis? A - Cryptosporidiosis B - Salmonellosis C - Bovine Viral Diarrhea, mucosal form D - Coccidiosis E - Ostertagiasis

B - Salmonellosis If you see "pseudo-diphtheritic membranes" in the intestines, think Neonatal salmonella. Look also for fever, diarrhea, especially with a "Septic tank odor" and systemic signs like septicemia, pneumonia, meningitis. Can die suddenly without diarrhea. Other rule outs for OLDER calf diarrheas include: Coccidiosis ( Older than 21 days and in ALL ages, bloody diarrhea, tenesmus) Ostertagiasis (anorexia, poor growth, diarrhea in less than 2 year olds on pasture- a chronic disease) BVD (Young cattle 6-24 mos, with mucosal lesions, diarrhea) Winter dysentery (explosive outbreak watery diarrhea with clotted dark blood, stabled animals, all ages in winter) and Johne's disease (Intractable diarrhea, cachexia). Crypto is typically a disease of the young, 1-4 weeks old.

What is the proper name for the swine kidney worm? A - Capillaria plica B - Stephanurus dentatus C - Dioctophyma renale D - Gnathostoma spinigerum E - Trichinella spiralis

B - Stephanurus dentatus Found worldwide, particularly in tropical and subtropical areas. Seen in the U.S. as a parasite of pigs raised outside in the southeastern and southcentral states.

Which one of the following vaccines is considered a core vaccine for South American camelids? A - Rabies B - Tetanus C - Listeria D - Equine herpes virus I E - Bovine viral diarrhea virus

B - Tetanus Tetanus and Clostridium perfringens type C and D toxoids are the core vaccines recommended in South American camelids. Rabies vaccination is indicated in rabies-endemic areas. Use of the vaccine for EHV-I in camelids (when exposed to exotic equids) has caused blindness and should only be used when necessary. Vaccination for BVDV may provide an antibody response and protection when exposed. No vaccines are licensed for use in camelids.

An obese 6 year-old neutered male Pomeranian presents with a 2-year history of intermittent dry, honking cough. This morning he started retching. The dog never boards in a kennel and has not traveled recently. On physical exam, a cough can be elicited by pressing on the trachea. A heart murmur is noted, loudest on the left at the 5th-6th intercostal space. The rest of the exam was unremarkable. What diagnosis is at the top of the differential list? Value Normal T=102.2 F (39 C) N=99.5-102.5 F, N=37.2-39.2 C HR=120 bpm N=60-120 RR=80 brpm N=15-34 CRT=2 sec N= less than 2 sec MM=pink A - Congestive heart failure B - Tracheal collapse C - Chronic bronchitis D - Laryngeal paralysis E - Infectious tracheobronchitis

B - Tracheal collapse When you hear a "HONKING COUGH" and heart murmur in a fat little dog that coughs after touching his tracheal area, think tracheal collapse. Toy/Min. breeds (Pomeranians, Toy poodle, Min. Poodle, Yorkies) are predisposed. Mitral valve insufficiency/dysplasia murmurs are often detached concurrently. Intermittent hx and lack of fever suggest a chronic, non-infectious cause. For definitive diagnosis do thoracic radiographs, bronchoscopy.

Suzie-Q, a 6-month old female spayed domestic short hair was recently adopted from the humane society. She has had watery diarrhea since adoption. Her fecal float and Giardia ELISA tests were negative. She was treated with metronidazole with no clinical improvement. You soak a cotton tip swab with saline and swab the rectum. You see elongated motile oval shaped protozoan organisms that do not look like Giardia lamblia. What organism might this be and what is the appropriate therapy? A - Paragonimus kellicotti, Praziquantel B - Tritrichomonas foetus, Ronidazole C - Enterobius vermicularis, Fenbendazole D - Cryptosporidium, Clindamycin E - Taenia taeniaformis, Praziquantel F - Giardia intestinalis, Fenbendazole

B - Tritrichomonas foetus, Ronidazole Tritrichomonas foetus is a flagellated parasite most commonly found in kittens that have had an unresponsive diarrhea. The parasite can be very difficult to diagnose. It is most often responsive to Ronidazole. Paragonimus is a lung fluke. The eggs are typically passed in the feces. Fenbendazole and Praziquantel have been effective against this parasite. Giardia is unlikely if the ELISA is negative as it is a very sensitive test. Taenia is a tapeworm and is not a flagellated parasite. It is treated with Praziquantel. Cryptosporidium is a coccidian that invades the small intestinal villi after ingestion of infected oocysts. It can be diagnosed with PCR. It is treated with clindamycin, azithromycin, or tylosin most commonly. It is usually an opportunist, so evaluation for underlying disease is appropriate. Enterobius vermicularis, or pinworm, is a parasite of people and primates but not cats and dogs.

A 2-year old Quarter Horse presents for intermittent muscle fasciculation followed by weakness. What condition should you suspect? A - Myotonia B - Hyperkalemic periodic paralysis C - Stringhalt D - Tetanus E - Grass tetany

B- Hyperkalemic periodic paralysis The correct answer is hyperkalemic periodic paralysis (HYPP). HYPP is seen in Quarter Horses due to a point mutation in a key part of a skeletal muscle sodium channel subunit. This results in elevation of the resting membrane potential to increase the likelihood of depolarizing. Excess concentrations of potassium can result in failure of the sodium channels to inactivate. Therefore, treatment is directed at decreasing dietary potassium.

A 6-year old male neutered terrier mix weighing 10kg presents to you after having ingested 3 pieces of sugar-free gum containing xylitol. What bloodwork abnormality are you expecting to find? A - Hypocalcemia B - Hyponatremia C - Hypoglycemia D - Hypernatremia E - Hyperglycemia

C - Hypoglycemia Xylitol is a sugar alcohol present in sugar-free gum that, when ingested, causes a rapid release of insulin in dogs. After ingestion, clinically significant hypoglycemia can develop within 30 minutes and can last for more than 12 hours. Acute hepatic necrosis and failure after higher doses of xylitol ingestion can occur as well.

Which lesions are most commonly associated with Marek's disease? A - Edematous facial swelling with sinusitis B - Hemorrhagic skin lesions C - Nerve enlargement D - Wrinkled eggs E - Blood in the trachea

C - Nerve enlargement Think of NERVE ENLARGEMENT with Marek's disease. Also, distortion of the pupil, enlargement of feather follicles ("skin leukosis"=condemnation of carcass). May see one leg forward, one leg back, a transient paralysis. Think infectious laryngotracheitis (ILT) if there is blood occluding trachea on necropsy. Remember wrinkled eggs go with infectious bronchitis. Think infectious coryza with sinusitis, swelling under eyes.

Which of these are used in the minor cross match for blood products when looking for a compatible blood donor for a dog? A - Donor red blood cells, recipient plasma B - Donor red blood cells, donor plasma C - Recipient red blood cells, donor plasma D - Recipient red blood cells, recipient plasma

C - Recipient red blood cells, donor plasma In a minor crossmatch, you are looking to see if the factors in the plasma of the donor are going to react to the recipient's red blood cells.

A 2 year-old budgie (parakeet) is presented with a history of limping on his left leg, which has gotten progressively worse over the last 3 weeks. He is eating and drinking normally, but has fallen off his perch twice in the last two days. Weakness in the left leg compared to the right one is observed. He retracts his left more slowly and more weakly when his toe is pinched. What condition is at the top of the differential diagnosis list? A - Pacheco's disease B - Polyoma virus infection C - Renal tumor D - Trauma- broken leg E - Hypovitaminosis A

C - Renal tumor Think renal tumor with progressive unilateral lameness in budgies (and poor prognosis). The sciatic nerve passes through the renal parenchyma in birds, and budgies are prone to neoplasia in general. Avian Polyomavirus is a YOUNG BIRD disease characterized by lethargy, crop stasis, death, and sometimes SUBQ hemorrhages after injections. Pacheco's disease - think "sudden death", younger birds. Hypovitaminosis A - think sinusitis, "bug-eyes" (exopthalmia), sublingual abscesses, white plaques. Broken bone possible, but would expect acute onset, not progressive.

A 6-year old Standardbred jumper is presented with a 3-month history of poor performance and intermittent shifting hindleg lameness. On physical exam, there is poor muscling of the gluteal muscles and some asymmetry to the croup (rump). The horse has a 7 cm. swelling over the hindquarters on the left dorsal side and shows pain and a reluctance to ventroflex the back when midline pressure is applied. On rectal palpation crepitation and shifting can be felt dorsally as the horse walks slowly forward. What is the diagnosis? A - Croup myopathy (longissimus dorsi, supraspinous ligament) B - Coxofemoral luxation C - Sacroiliac subluxation D - Overlapping vertebral spinous processes E - Fibrotic ossifying myopathy

C - Sacroiliac subluxation Think of sacroiliac subluxation or sprain in an athletic jumping horse with - intermittent hindlimb lameness localized in the croup (rump) - evidence of back pain - swelling over the tuber sacrale (hunter's bumps) - crepitance felt rectally in the sacroiliac area Horses with hunter's bumps can be sound if the injury has completely healed, but lameness can be observed if the injury is recent or has never resolved. Sometimes confused with stifle lameness. Coxofemoral luxation is rare in horses due to a deep acetabulum and the presence of the accessory ligament of the hip, unique to horses. Overlapping vertebral spinous processes is a problem of thoracic and lumbar vertebra under the saddle area, seen in short-backed eventing horses (jumpers/hunter, dressage). Back muscle and ligament strain is the #1 cause of back pain in horses, accounting for 33% of cases. Most commonly associated with the longissimus dorsi muscle, sublumbar illiopsoas muscles and the supraspinous ligament and also seen in eventing horses.

During the necropsy of an 8 year-old mixed breed dog from the Southern United States, reactive granulomas in the esophagus containing bright red worms, 40 mm to 70 mm long are found. What is the diagnosis? A - Ollulanus tricuspis B - Haemonchus placei C - Spirocerca lupi D - Gastrophilus spp. E - Physaloptera spp.

C - Spirocerca lupi A disease of dogs in the Southern U.S. and tropical climates, Spirocerca lupi (esophageal worms) make reactive granulomas of variable size in the esophageal, gastric or aortic walls. Spirocercosis may also lead to aneurysm in the thoracic aorta or an ossifying spondylitis of the posterior thoracic vertebrae. Typically asymptomatic, but large granulomas can cause esophageal obstruction. Large granulomas may become neoplastic (osteosarcoma, fibrosarcoma). Some dogs develop spondylitis or enlargement of the extremities characteristic of hypertrophic osteopathy. All four of the other choices are gastric parasites. Ollulanus tricuspis is an uncommon gastric parasite of cats. Physaloptera spp. is a stomach worm that may cause vomiting, anorexia, dark feces in dogs and cats. Haemonchus spp., Ostertagia spp. and Trichostrongylus spp. are found in the abomasum of ruminants. Gastrophilus spp. are the larvae of horse bot flies, found in the stomach of horses.

An unweaned 2-month old calf is presented for necropsy. The calf collapsed and suddenly died after she escaped and was chased around the back pasture for 45 minutes by her owner. Among other things, white myocardial and endocardial streaking in the left ventricle of this calf's heart are evident. What advice should be given the farmer? A - Put all the calves off ionophore-containing feed B - Start the other calves on Ceftiofur (Naxcel®) C - Treat the other calves with vitamin E/Selenium D - Check the mother for bovine leukosis by AGID E - Search the calf barn for sources of lead

C - Treat the other calves with vitamin E/Selenium. Think of white muscle disease when you see SUDDEN DEATH and endocardial PLAQUES in a young calf, lamb or kid with a history of recent VIGOROUS EXERCISE. Typically seen in young, fast-growing animals (ie: calves 2 weeks-6 mos). Clinical signs may include dyspnea (due to myocardial disease), stiff gait, arched back, weakness, recumbent but BAR (bright, alert, responsive). Sudden death may resemble enterotoxemia, should see acute bloody diarrhea, convulsions, opisthotonos in first days of life with enterotoxemia.

A 6 year old female spayed golden retriever is presented for acute onset of flaccid jaw paralysis. The dog is unable to close her mouth, has difficulty eating and is drooling. There is no history of trauma. What is the most likely diagnosis? A - Masticatory myositis B - Facial nerve paralysis C - Trigeminal neuritis D - Myasthenia gravis E - Meningoencephalitis

C - Trigeminal neuritis Idiopathic trigeminal neuritis, cranial nerve 5, (CN 5) is characterized by acute onset of flaccid jaw paralysis. Affected animals cannot close their mouth and have difficulty eating and drinking. In some cases unilateral or bilateral Horner's syndrome, facial paresis, and decreased facial sensation are present. Most common in dogs, rare in cats. Cause is unknown. Idiopathic facial nerve paralysis, cranial nerve 7 (CN 7) results in the inability to move the eyelid, lip or ear and dryness of the eyes and mouth. Masticatory myositis, an immune-mediated disease of dogs, is characterized by pain on opening the mouth and swelling of the muscles of mastication (acute) or atrophy of the temporalis and masseter muscles with the inability to open the mouth due to fibrosis (chronic). Myasthenia Gravis (MG), a neuromuscular disease, typically presents with episodic or exercise-induced weakness due to impaired transmission of acetylcholine at the neuromuscular junction of skeletal muscles. Meningoencephalitis, inflammation of the brain and meninges, should always be on the differential list. Lesions may be anywhere in the CNS and may present with any neurological signs.

When performing a subtotal colectomy on a feline patient, what blood vessel limits the amount of colon that you are able to remove? A - Pudendal artery B - Left colic artery C - Ileocolic artery D - Caudal mesenteric artery

C - ileocolic artery The site for colonic resection is limited by tension on the ileocolic artery when trying to suture your new end of colon to the rectum. Sometimes the tension is too great and instead of a colocolic anastomosis, an ileocolic anastomosis must be performed. Essentially you are trying to connect a section of ascending colon to the rectum. Performing an ileocolic anastomosis is not ideal because you eliminate the ileocecal valve, and that may predispose the animal to bacterial overgrowth. The caudal mesenteric artery gives branches to the rectum and descending colon. The left colic artery also feeds the descending colon. The pudendal artery supplies the external genitalia. The ileocolic artery provides blood supply to the ascending and transverse colon.

A 5-year old male castrated Mastiff presents for left pelvic limb lameness. The medial aspects of both stifles are thickened. Manipulation of the left stifle reveals cranial motion of the tibia relative to the femur and a clicking sound from the joint on flexion and extension. What is the most likely diagnosis? A - Left cranial cruciate ligament rupture with no meniscal cartilage tear B - Left luxating patella C - Left cranial cruciate ligament rupture with meniscal cartilage tear D - Left caudal cruciate ligament rupture with no meniscal cartilage tear

C - left cranial cruciate ligament rupture with meniscal cartilage tear The cranial motion of the tibia and medial thickening of the joint (also known as medial buttress) is consistent with the commonly torn cranial cruciate ligament. The clicking heard on flexion and extension is consistent with damaged medial meniscus cartilage found in the stifle.

You are asked to examine some feeder pigs that have stopped eating yesterday. The group is lying down and seems lethargic. They have fevers of 105-106F (40.6 -41.1 C), firm dry feces, and the skin has rhomboid-shaped red blotches scattered on it. What treatment should be recommended? A - Gentamicin B - Streptomycin C - Penicillin D - Chloramphenicol E - Metronidazole

C- Penicillin Erysipelas is susceptible to penicillins, as well as tetracyclines (usually), lincomycin and tylosin. Chloramphenicol and nitroimidazoles (including metronidazole) are not approved for food animal use. Disease outbreaks may be acute or chronic; clinically inapparent infections also occur. Acute outbreaks are characterized by sudden and unexpected deaths, febrile episodes, painful joints, and skin lesions that vary from generalized cyanosis to the often-described diamond skin (rhomboid urticaria) lesions. Chronic erysipelas tends to follow acute outbreaks and is characterized by enlarged joints and lameness. A second form of chronic erysipelias is vegetative valvular endocarditis. Pigs with valvular lesions may exhibit few clinical signs, however, when exerted physically they may show signs of respiratory distress, lethargy, and cyanosis, and possibly suddenly succumb to the infection.

An immunochromatographic test kit for detection of fecal canine parvoviruses (CPV) antigen is being tested in a local cat shelter where as many as 10% of the cats there may have panleukopenia secondary to infection with the canine parvovirus. Here are simulated test results, compared to a gold standard test for CPV. CPV pos CPV neg Total Test kit positive 128 734 862 Test kit negative 63 1575 1638 Total 191 2309 2500 What is the sensitivity of this test kit? A - 734/862 B - 128/734 C - 1575/2309 D - 128/191 E - 1575/1638

D - 128/191 Sensitivity=128/191 (67%) a/(a+c) This is a classic example of a 2x2 table in epidemiology, used to compare a new test (the immunochromatographic test kit) to a gold standard test (the CPV test). Draw a 2x2 table, and label the boxes a,b,c,d. Sensitivity = a/(a+c). Click here to see a Basic 2X2 table. Feline panleukopenia virus (FPV) is closely related to type 2 canine parvoviruses (CPV-2, CPV-2a, CPV-2b). CPV-2a and CPV-2b have been shown to cause a panleukopenia-like illness in domestic cats.

Tommy, a 4-year old male neutered domestic short hair, presents to you for frequent urination. Urinalysis reveals 4+ struvite crystals. An abdominal radiograph shows a 1 cm round calculus in the bladder. Tommy does not have a urethral blockage, and urine culture is negative. The owner says surgery to remove this stone is not an option due to finances. Which of the following would be the most important treatment for Tommy? A - Metacam B - Potassium citrate C - Clavamox D - Acidifying diet E - Alkalinizing diet

D - Acidifying diet This cat most likely has a struvite bladder stone. This cannot be determined without a stone analysis, but based on the signalment of the patient, the crystalluria, and the radiograph, this is the most likely type. These types of stones typically form in urine with a high pH. Therefore, an acidifying diet would be appropriate for stone dissolution. Calcium oxalate stones are the most common type of stone to form in acidic urine. For this type, an alkalinizing diet would be the most appropriate. Clavamox would only be indicated if this cat had a urinary tract infection sensitive to this antibiotic. This cat had a negative urine culture. Metacam can be used short-term to help with pain and inflammation associated with the cystitis from this condition but unfortunately Metacam is no longer recommend for use in cats in the United States. Potassium citrate is a supplement that can be given in addition to an alkalinizing diet to prevent the formation of calcium oxalate stones. This would be contraindicated in this case.

In September, a 15-year-old Quarterhorse mare is presented with a 5-day history of left head tilt, facial paralysis, depression, and stumbling. The horse is ataxic and knuckles on both forelegs, worse on the left. There is muscle atrophy of the left pectorals and right hindquarters and strips of localized spontaneous sweating over the left trunk. A serum:CSF ratio is consistent with a diagnosis of Sarcocystis neurona. What prognosis is given to the owner if she elects to treat the horse? A - Only 25% of treated horses improve in their neurologic state B - 50% of treated horses recover completely C - Prognosis is good if treated with doxycycline D - At least 60% of treated horses improve E - Prognosis is grave

D - At least 60% of treated horses improve. The prognosis is guarded to fair for treatment of equine protozoal myeloencephalitis (EPM). Less than 25% recover completely, but at least 60% of treated horses improve. Affected animals typically improve by 1-2 ataxia grades (on the Mayhew ataxia scale of 1-5). The earlier treatment is started, the better the outcome. 10-20% of horses relapse within 2 years of initial diagnosis.

A 4 year old mare has bilateral swelling and drainage of the mandibular lymph nodes. Rectal temperature is 101.8F (38.8 C). Which of the following is the most appropriate plan for this horse? A - Administer penicillin B - Administer corticosteroids C - Inform the state veterinarian D - Culture the discharge for bacteria

D - Culture the discharge for bacteria The most likely diagnosis for this horse is equine strangles. Strangles most commonly affects younger horses (<5 years of age), but can cause disease in any age horse. The etiologic agent of this disease is Streptococcus equi subsp. equi. The diagnostic test of choice to confirm this is bacterial culture. While awaiting culture results, the horse should be separated from any other horses, as strangles is highly contagious to other horses. Antibiotic therapy is controversial and thought to lengthen the course of disease rather than shorten it when given at this stage; also, it may possibly interfere with the natural immunity acquired from natural infection. Strangles is a reportable disease.

Which of the following drugs is known for stimulating appetite in small animals? A - Tetracycline B - Acyclovir C - Omeprazole D - Cyproheptadine

D - Cyproheptadine Other appetite stimulants used for stimulating food consumption include mirtazipine, diazepam and oxazepam. Omeprazole is a proton pump inhibitor indicated for gastric ulcers in horses and small animals. Tetracycline is an antibiotic that inhibits protein synthesis. Acyclovir is an antiviral indicated for herpesvirus.

Which of the following drugs is known for stimulating appetite in small animals? A - Omeprazole B - Tetracycline C - Acyclovir D - Cyproheptadine

D - Cyproheptadine The correct answer is cyproheptadine. Other appetite stimulants used for stimulating food consumption include mirtazipine, diazepam and oxazepam. Omeprazole is a PPI used for gastric ulcers in horses and small animals. It irreversibly inhibits H/K ATPase at secretory surface of parietal cells Tetracycline is a broad-spectrum antibiotic. It reversibly binds to 30S subunit to prevent attachment of aminoacyl-tRNA to RNA-ribosome complex. Acyclovir is an antiviral drug that inhibits the replication of herpesviruses.

A 1.5-year-old spayed female cat presents with a 7-day history of vocalizing, rolling, and allowing a male neutered cat in the household to mount her. The cat has gone through one similar episode 1 month ago. She otherwise has been behaving normally and has no other health problems. You performed an ovariohysterectomy on the cat at 3 months of age. You perform vaginal cytology which shows some cornified epithelial cells but is inconclusive. You measure serum lutenizing hormone of 0.2 ng/ml (normal for an ovariectomized female is >1 ng/ml). Which of the following is the most appropriate course of action? A - Measure serum estrogen and progesterone levels B - Order MRI of the brain to rule out a pituitary tumor C - Measure serum testosterone levels D - Exploratory laparotomy to remove the ovarian remnant

D - Exploratory laparotomy to remove the ovarian remnant This is a classic description of ovarian remnant syndrome, which is when a cat goes into estrus after previously having an ovariohysterectomy (OVH). This can occur anywhere from weeks to years after OVH and typically the clinical signs consistent with estrus are sufficient to conclude that the cat is in estrus and has ovarian tissue present. Additional diagnostic tests that are consistent with ovarian remnant syndrome include: Serum estrogen >70 pmol/L indicate that the cat has estrogen production from the ovary. The problem with this test is that estrogen measurements may fluctuate and can be unreliable. Serum progesterone >6 nmol/L after induced ovulation is sufficient to conclude that corpora lutea formed and released progesterone. Testing for serum LH levels can also help confirm the diagnosis. In intact queens, LH is consistently maintained at basal levels due to negative feedback from ovarian estradiol secretion. After OVH, this control is lost and LH concentrations increase. LH <1 ng/mL is consistent with the presence of an ovary as it is in this case. Surgery is the treatment of choice. Many practitioners prefer to do surgery during estrus or diestrus when the ovarian tissue is enlarged and easier to locate. Remnants may be bilateral so a complete exploratory laparatomy is necessary. Ovarian tissue is most commonly at the ovarian pedicle but can also be in the mesentery or elsewhere. You should be aware that ovarian remnant syndrome is NOT associated with new graduates or inexperienced surgeons (it is not likely the fault of the initial surgeon). It is suspected that accessory ovarian tissue separate from the ovary within the ovarian ligament or proper ligament of the ovary may become functional and cause this condition in most cases.

In which one of the following cattle breeds is syndactyly most commonly inherited? A - Brown Swiss B - Angus C - Hereford D - Holstein-Friesian E - Simmental

D - Holstein-Friesian Holstein-Friesian cattle inherit syndactyly, or "mule foot", a simple autosomal recessive trait, more often than other breeds. Syndactyly is the partial or complete fusion of the digits of one or more feet. Forefeet are affected most often. Affected animals walk slowly, usually with a high-stepping gait. They may also be more prone to hyperthermia than normal cows.

Which choice describes inflammation of an acquired bursa over the dorsal carpus in the horse? A - Villonodular synovitis B - Ulceroproliferative stomatitis C - Atheroma D - Hygroma E - Suspensory desmitis

D - Hygroma A hygroma is inflammation of an acquired bursa that develops where normally there is no bursa due to trauma to the dorsum of the carpus. In dogs, you frequently see hygromas at bony, unpadded areas like the elbow. Villonodular synovitis is an inflammation of the synovial membrane of the dorsoproximal aspect of the forelimb fetlock joints. Ulceroproliferative stomatitis is a progressively worsening gingivitis and stomatitis in cats. The glossopalatine arches often have severely ulcerated, friable, inflamed, and proliferative lesions. A sebaceous cyst in the dorsolateral aspect of the nasal diverticulum (false nostril) is an atheroma. In humans, an atheroma usually refers to a fatty deposit in the intima (inner lining) of an artery, resulting from atherosclerosis.

In which equine virus is antigenic drift the most important in the development of novel strains? A - Adenovirus B - Herpesvirus-4 C - Herpesvirus-1 D - Influenza virus E - Papillomavirus

D - Influenza Virus Antigenic drift is most important in the epidemiology of equine influenza. Point mutations in the genes for hemagglutinin and neuraminidase cause changes in these surface proteins that can lead to the development of new virus strains not recognized by antibodies to earlier strains. Because influenza is an RNA virus without proofreading ability, mutations can occur more rapidly with flu than mutations seen in DNA viruses like equid herpesviruses and adenoviruses.

A 2-year old heifer is presented who died the night before. She was stunted compared to her herdmates, and had a 3-week history of poor appetite, unthriftiness, diarrhea. Necropsy reveals that the abomasum is edematous and covered in small umbilicated nodules 1-2 mm in diameter (cobblestone or "Moroccan leather" appearance). Several other younger cows in this Spring-calving herd look unthrifty and have diarrhea too. Which one of the following options is the most consistently effective against all stages of this parasite? A - Amprolium B - Thiabendazole C - Levamisole D - Ivermectin E - Morantel

D - Ivermectin Avermectins (macrocyclic lactones), such as ivermectin, are the most consistently effective of these choices against all stages, including hypobiotic larvae, of Ostertagia, the parasite in this calf's gastrointestinal tract. An abomasum with a cobblestone or "Moroccan leather" appearance is pathognomonic for Ostertagia , one of the three stomach worms of cattle. In order to minimize the development of anthelmintic resistance, current recommendations for parasite control include not only appropriate deworming, but adequate nutrition and rotation of treated stock to "clean" pastures. Some experts recommend deworming of only the most severely affected animals in a group (targeted treatment). The idea is to slow development of drug-resistant populations by allowing some worms to survive (these are called "refugia"). If some worms survive that are susceptible to antiparasitic drugs, it DECREASES population pressure to develop drug resistance. Amprolium is your treatment of choices for calves and cows with coccidiosis.

For the last 10 years, your state has had a mandatory vaccination program against "pedunculated giblet disease" in fur-bearing turtles and the prevalence of this terrible disease has decreased markedly. How does this decrease in prevalence affect the predictive value positive (PVP) of the best serologic test for pedunculated giblet disease? A - PVP depends on the number tested, not prevalence B - PVP is affected by specificity, not prevalence C - PVP increases as prevalence decreases D - PVP decreases as prevalence decreases E - PVP stays the same as prevalence decreases

D - PVP decreases as prevalence decreases Don't think too hard on this: As prevalence of a disease goes DOWN, PVP of your serologic test also goes DOWN. That is, as your disease becomes more and more rare, the predictive value of your same old test gets WORSE. That is all you need to know. Read more only if you want to see the math, (but you don't need it.) Lets say prevalence of pedunculated giblet disease is 30% in 1000 turtles (that's 300 infected, 700 disease-free, then). A test with 90% sensitivity would correctly Dx 270 (cell "a") with the disease (true pos) and Correctly say 30 were negative (cell "c"); a test with 90% specificity would correctly Dx 630 (cell "d") as disease-free (true negs) and Correctly say 70 were positive (cell "b"): PVP=a/(a+b)=270/(270+70)= 79%. Now, do the same math on 1000 turtles, but assume prevalence is now only 1% (0.01) after your vaccination program. You will see PVP goes down to 47%.

Failure to control mild endemic respiratory disease in swine caused by Mycoplasma hyopneumoniae predisposes pigs to complications. Exudative bronchopneumonia and polyarthritis are most commonly seen in herds infected with both Mycoplasma hyopneumoniae AND which other organism? A - Bordetella bronchiseptica B - Fusobacterium necrophorum C - Hemophilus parasuis D - Pasteurella multocida E - Swine influenza virus

D - Pasteurella multocida Pasteurella multocida infection in conjunction with Mycoplasma hyopneumoniae causes exudative bronchopneumonia, polyarthritis, and chronic lung lesions. Mycoplasma hyopneumoniae, (also called "Enzootic pneumonia") is a common, smoldering low-level illness. Stressors (parasites, other infections, even weather) can result in severe pneumonia. Best managed by decreasing stressors with improved ventilation and decreased overcrowding. In endemic herds, ABX for sick individuals (ie: lincomycin, tylosin, tiamulin, or a tetracycline) helps control illness, most likely by preventing secondary bacterial infection (like Pasteurella). Bacterin vaccines give good protection, decrease signs (coughing). Pre-farrowing vaccination of sows decreases colonization in suckling piglets.

Which of the following is the causative agent of fowl cholera? A - Mycoplasma gallisepticum B - Reticuloendotheliosis virus C - Chlamydophila psittaci D - Pasteurella multocida

D - Pasteurella multocida The correct answer is Pasteurella multocida. The bacterium is a gram negative rod. It causes fever, mucoid discharge from the mouth, diarrhea, petechia, ecchymoses, increased pericardial and peritoneal fluid, and death. The chronic form of the disease usually causes localized disease. Diagnosis is based on identifying the organism from samples in conjunction with clinical signs. Vaccines are available for prevention of the disease. Sulfa antibiotics are used for treatment. Mycoplasma gallisepticum causes chronic respiratory disease of chickens and infectious sinusitis of turkeys. Chlamydophila psittaci causes psittacosis

A 3-year old castrated male Doberman Pinscher presents for acute onset non-productive vomiting of 3 hours duration. On exam, the patient is extremely lethargic, tachycardic, has weak pulses and a distended abdomen. Following standard initial emergency treatment for suspected gastric dilation and volvulus, you would confirm the diagnosis with what imaging procedure? A - Ventral recumbency abdominal radiograph B - Dorsal recumbency abdominal radiograph C - Abdominal ultrasound exam D - Right lateral recumbency abdominal radiograph E - Barium contrast abdominal radiograph F - Left lateral recumbency abdominal radiograph

D - Right lateral recumbency abdominal radiograph A typical GDV occurs with repositioning of the pylorus to the left dorsal abdomen. A right lateral radiographic image is the best position for revealing a gas filled left dorsally-displaced pylorus with a gas-filled ventral fundus separated by a soft tissue band (compartmented stomach). Although the features of a malpositioned stomach may be observed on the other radiographic views, they are often more difficult to interpret than that represented by the right lateral recumbency view. The severe amount of gas within the GI tract that is associated with GDV would make ultrasound exam interpretation extremely difficult at best.

A 7 year-old cat is presented with miosis, ptosis, enophthalmos, and protruded nictitating membrane involving the right eye. These symptoms are caused by damage to which one of the following innervation pathways? A - Right trigeminal innervation to the eye B - Right parasympathetic innervation to the eye C - Left oculomotor innervation to the eye D - Right sympathetic innervation to the eye E - Left vagal innervation to the eye

D - Right sympathetic innervation to the eye. Horner's syndrome (miosis, ptosis, and enophthalmos) is caused by defective ipsilateral sympathetic innervation to the affected eye. Loss of parasympathetic innervation (via the oculomotor nerve) to the eye would cause a dilated pupil. Trigeminal damage would result in decreased sensation to the eye.

A cow is presented on emergency with urea/non-protein nitrogen toxicity. What is the treatment of choice? A - Rumenotomy B - Relieve bloat, drench with 2-8 liters sodium bicarbonate C - IV Fluids with MgSO4, Na thiosulfate PO D - Rumenal infusion 2-8 liters vinegar,3-10 gallons cold water E - Atropine, Protopam chloride IV q 4-6 hours

D - Rumenal infusion 2-8 liters vinegar, 3-10 gallons cold water Treat Urea/non-protein nitrogen (NPN) toxicity with a rumenal infusion 2-8 liters 5% acetic acid (vinegar) and 3-10 gallons of cold water. The vinegar decreases rumenal pH which slows absorption of un-ionized ammonia. Repeat Q 6 hours up to 48 hours. Best results if animal is still ambulatory. It is often impossible to treat these cases before they die because of rapid progression to death. If possible, Rx with IV fluids. If necessary, relieve bloat. Urea/NPN toxicity is related to ammoniated feed toxicity which causes so-called "Bovine bonkers". Manage ammoniated feed toxicity by removing the ammoniated feed and treating severe cases with anticonvulsants like diazepam, pentobarbital.

A one-month old foal develops fever, icterus, and diarrhea acutely. Bloodwork shows hyperfibrinogenemia, hypoglycemia, and elevated liver enzymes. Which of these conditions is most likely? A - Clostridium novyi type B B - Herpesviral hepatitis C - Theiler's disease D - Tyzzer's disease

D - Tyzzer's disease This is the most likely cause because of the age of the foal and the acute nature of the disease. Tyzzer's disease is caused by Clostridium piliformis, which causes an acute necrotizing hepatitis. It affects only foals from about 1-6 weeks of age. Theiler's disease is a condition of adult horses. Clostridium novyi is rare in horses and seen much more in sheep and cattle. Herpesvirus can cause hepatitis but is usually seen at or very soon after birth.

What are ongoing losses in fluid therapy? A - Urine output B - Fluids given minus extracellular volume estimate C - The tears I shed when I can't make sense of fluid calculations D - Vomit and diarrhea E - Percent dehydration multiplied by body weight

D - Vomit and diarrhea Ongoing losses are typically vomiting, diarrhea or other losses into a third compartment (pleura, abdomen etc). These are sometimes called "contemporary" losses. A classic example of a disease with significant ongoing losses is a puppy with parvovirus who has both diarrhea and vomiting. Sensible losses are easily measured, like the fluids the animal loses by urination. Insensible losses are the fluids an animal loses that are hard to measure, through breathing and via the skin. In humans and animals that sweat (like horses) sweating is part of insensible losses. Remember that all three types of loss must be accounted for (or at least estimated) when calculating fluids needed to rehydrate a dehydrated animal.

A 3-year-old Quarter horse/Appaloosa cross mare is presented on emergency because she collapsed in the last 30 minutes. The mare is recumbent and weak, with respiratory stridor, muscle fasciculations, sweating, prolapse of the third eyelid and her lips pulled back spasmodically, like a grin. The horse is hydrated and has had no recent history of colic. During the examination, she improves markedly and within 60 minutes stands and appears almost normal again with no treatment. What is the clinical diagnosis? Value Normal 100.2 F (37.9 C) 99.0-100.6, 37.2-38.1 C HR=24 bpm 28-40 bpm BR=16 brpm 10-14 brpm A - Myasthenia gravis B - Epilepsy C - Ryegrass staggers D - Hyperkalemic periodic paralysis E - Equine degenerative myelopathy

D. Hyperkalemic periodic paralysis This is likely to be hyperkalemic periodic paralysis (HyPP), an autosomal dominant trait found in ~ 4% of Quarter horses. Heterozygotes often have periodic episodes 15-90 min: prolapse 3rd eyelid, muscle fasciculations, collapse. 25% cases traced back to one stallion named "Impressive." Onset of signs unpredictable, and many triggers: diets high in POTASSIUM (>1.1%, ie: alfalfa hay, molasses, electrolyte supplements, kelp-based supplements). Fasting, anesthesia, heavy sedation, trailer rides, and stress can precipitate clinical signs. For ryegrass staggers think of tremors, incoordination, collapse. For equine degenerative myelopathy (EDM) think of ataxia in foals 6-8 mos. Epilepsy is poorly documented in horses. Myasthenia gravis basically a dog disease.

Which of these statements about pasteurellosis in rabbits is most accurate? A - 2-week course of antimicrobials is effective in eliminating disease B - It is a gram-positive, nonmotile coccobacillus C - Every positive test result indicates a pathologic condition D - Most common presentation is chronic, insidious arthritis E - 30-90% of adults in colonies are asymptomatic carriers

E - 30-90% of adults in colones are asymptomatic carriers 30-90% of adults in colonies are asymptomatic carriers of Pasteurella multocida so not every positive nasal swab means pathology. Like shipping fever in cattle, pasteurellosis in rabbits is multifactorial: stress (crowding, kindling) can precipitate disease and other organisms can be isolated from clinical cases. (Pseudomonas spp., Bordetella bronchiseptica, Staph spp.and Strep spp.). P. multocida is a gram-negative, nonmotile coccobacillus. Therapy is difficult and even long-term antibiotics often do not completely eliminate the bacteria.

A practice is using an FeLV test with a sensitivity of 90% and a specificity of 95%. Assuming the prevalence of feline leukemia in the area is 5%, what is the predictive value positive (PVP) of the test? A - 45% B - 90% C - 55% D - 88% E - 48%

E - 48% The trick with this kind of question is to pick an imaginary number of animals that you test, like 1000, and fill out your 2x2 table from there. If prevalence is 5% then there myust be 50/1000 cats with FeLV and 950 cats that are disease-free. A 90% sensitive test will correctly call 45/50 positive (box "a") and *IN-correctly* call 5/50 negative (box "c"). If 50/1000 animals are infected, then 95/1000 are disease-free. Your 95% specific test will correctly call 902/950 disease-free (box "d": 0.95x950=902) and IN-correctly call 48/950 positive. Now your a,b,c,d boxes are filled, it is easy to calculate PVP: a/(a+b)=45/(45+48)=48%

A 10-year old male castrated cat that you have previously diagnosed with hyperthyroidism presents to you for acute onset of blindness. You perform an ophthalmic exam and note retinal hemorrhage. What diagnostic test should you perform first? A - Serum BUN and creatinine B - Coagulation times C - Total T4 levels D - Free T4 levels by equilibrium dialysis E - Blood pressure

E - Blood pressure The correct answer is blood pressure. Cats with hyperthyroidism are likely to develop hypertension. If this is severe enough (>180-200 mmHg systolic), they can be at risk for acute retinal detachment or hemorrhage resulting in blindness. Prompt resolution of the hypertension is critical to prevent further damage to the eye and other organs.

A stray dog is presented after being hit by a car. The dog has hypotonic forelimbs and spastic paresis in the hinds. All four limbs have proprioceptive deficits and sensation loss-signs are worse in the forelimbs. Where is the lesion? A - Cranial cervical: C1-C5 B - Cannot say without cutaneous trunci reflex results C - Lumbosacral L4-S3 D - Thoracolumbar T3-L3 E - Cervicothoracic: C6-T2

E - Cervicothoracic: C6-T2. Weak, hypotonic (Lower motor neuron-LMN) forelimbs and spastic paresis (Upper motor neuron-UMN) hindlimbs are signs of a cervicothoracic (C6-T2) lesion. May see worse signs in fores then hinds. NOTE: This is the OPPOSITE presentation as Schiff-Sherrington syndrome ie: SEVERE acute spinal cord trauma in the region of T3-L3, where in lateral recumbency, the thoracic limbs are rigid and extended and the pelvic limbs appear flaccid in comparison. However, pelvic limb reflexes are normal to increased, as would be expected with an upper motor neuron lesion. Can localize T3-L3 lesions by checking cutaneous trunci reflex - the lesion is usually 1-2 vertebrae cranial to where the reflex disappears. With C1-C5 would expect UMN signs in all 4 limbs, usually worse in hinds. With a T3-L3, would see UMN hind limb signs and normal forelimbs.

An adopted adult cat from a shelter with an unknown vaccination history tests positive for feline leukemia virus (FeLV) infection by IFA. What should the owner be told? A - Possible transient FeLV infection B - Need an ELISA test in 12 weeks to confirm diagnosis C - Cat may be vaccinated for FeLV D - Need a Western blot test to confirm diagnosis E - Strong chance of lifelong FeLV infection

E - Strong chance of lifelong FeLV infection 97% of cats positive for feline leukemia virus (FeLV) by IFA remain persistently infected and viremic for life. The FeLV ELISA is more seNsitive than the IFA (fewer false Negs, so trust a NEG test more), so it is a better general screening test with which to start. The FeLV IFA is more sPecific than the ELISA, (fewer false Pos, so trust a POS test more) so it is a better confirmatory test for any cats with a positive FeLV ELISA. THREE things to remember about FeLV and vaccinations: 1. FeLV ELISA and IFA tests measure ANTIGEN, not antibody, so FeLV vaccination does NOT interfere with testing. 2.Vaccinate FeLV-positive cats yearly against respiratory, enteric viruses with inactivated vaccines. 3. FeLV vaccination for FeLV-negative cats has been associated with sarcomas. Vaccinate LOW on the LEFT hind.

One description of a typical heart sound is "lub-dub" What makes the first heart sound? (the "lub" ) A - Ventricular filling B - Aortic and pulmonic valves C - Mitral and semilunar valves D - Atrial contraction E - Atrioventricular valves

E Atrioventricular valves The first heart sound (S1-the "lub" in "lub-dub") is caused by closure of the atrioventricular valves (AV valves, mitral and tricuspid). The second sound (S2) is the closure of the aortic and pulmonic valves (semilunar valves). A third sound (S3) is the rush of ventricular filling and a fourth sound (S4) is atrial systole (atrial contraction). You can often hear all 4 sounds in horses, but typically hear only S1 and S2 in cattle and small animals.


Conjuntos de estudio relacionados

Module 14: Nursing Care of Patients with Disorders of the Musculoskeletal System

View Set

Community ALA ATI- Study for exam

View Set

MSN 377: Ch 70 Workbook Questions

View Set

Social Studies Chapter 5 questions

View Set